The government will purchase and install new severe weather sirens for this area next year if replacement parts for t...

MJA7 on November 2, 2014

Question explanation help.

Please explain why correct answer is D. I reasoned correct answer was E since inferior quality would result in public being less safe.

Replies
Create a free account to read and take part in forum discussions.

Already have an account? log in

Naz on November 5, 2014

Here we have a strengthen with necessary premise question. Remember that a premise is necessary for a conclusion if the falsity of the premise guarantees or brings about the falsity of the conclusion. First we check to see if the answer choice strengthens the passage, and then, if it does strengthen, we negate the answer choice to see if its negation makes the argument fall apart. If the answer choice does both those things then it is our correct answer.

Conclusion: "the public will be safer during severe weather in the future."

Let's diagram.

"The government will purchase and install new severe weather sirens for this area next year if replacement parts for the old sirens are difficult to obtain."

P1: RPDO ==> GPNS
not GPNS ==> not RPDO

"The news paper claims that public safety in the event of severe weather would be enhanced if new sirens were to be installed."

P2: GPNS ==> PSE
not PSE ==> not GPNS

"The local company from which replacement parts were purchased last year has since gone out of business."

P3: LCOOB

"If the newspaper is correct, the public will be safer during severe weather in the future."

C: PSE

Answer choice (D): "Replacement parts for old sirens will be difficult to obtain if the government cannot obtain them from the company it purchased them from last year."

(D): LCOOB ==> RPDO
not RPDO ==> not LCOOB

Does this strengthen the argument? Yes. If (D) is true, we can connect it to P1 like so: LCOOB ==> RPDO ==> GPNS to infer: LCOOB ==> GPNS, which we can connect to P2 to conclude: LCOOB ==> PSE. Thus, since we know from P3 that we have LCOOB, we can conclude PSE, which is our conclusion. Therefore, answer choice (D) strengthens the argument.

Does the negation of answer choice (D) make the answer fall apart? Yes.

Negation: Replacement parts for old sirens will not necessarily be difficult to obtain if the government cannot obtain them from the company it purchased them from last year.

So, if this is true, then the conclusion would no longer necessarily follow because we can no longer infer that the government will most likely install new sirens which would enhance public safety .

Answer choice (E) is not the answer because we are concerned with replacement parts being difficult to obtain, not the quality of available replacement parts. All we know is that if replacement parts are difficult to obtain, then the government will install new sirens. We do not know what happens when there are replacement parts available, but their quality is not up to par. Thus, answer choice (E) does not strengthen.

Hope that clears things up! Please let us know if you have any other questions.

MJA7 on November 7, 2014

Thanks for the add'l info. From your explanation I believe it reiterates that strengthen questions are concerned with the relationship between the conclusion and support and not on deducing information not provided to us in the passage.

Naz on November 11, 2014

Exactly. Would you like to clarify what you mean by "deducing information not provided to us in the passage?"

We should never deduce from information not provided to us in the passage. The limit of our knowledge on the LSAT is within the boundaries of the stimulus. So, if it doesn't state it in the stimulus, it is not our answer.

Hope that helps! Please let us know if you have any other questions.

kuvimec on July 22, 2020

Why is this a necessary assumption question and not a sufficient assumption question. The stem sounds like a sufficient assumption question.

Khaled-Atallah on August 3, 2020

Same question, I got the answer right but isn't this a strengthen with sufficient?

Victoria on August 6, 2020

Hi @kuvimec and @Khaled-Atallah,

You are both correct - this is a Strengthen with Sufficient Premise question.

A Strengthen with Necessary Premise question will ask for an assumption that is required or that the argument is dependent upon and will use necessary indicators.

A Strengthen with Sufficient Premise question will ask for an assumption that, if provided, would allow the argument to be properly drawn and will use sufficient indicators, primarily "if."

Keep up the good work and please let us know if you have any further questions!